Your-Doctor
Multiple Choice Questions (MCQ)



Free Palestine
Quiz Categories Click to expand

Category: Cardiology--->Pharmacology
Page: 4

Question 16# Print Question

N. M. is a 75-year-old woman with a long-standing history of HF secondary to viral cardiomyopathy. She presents to the outpatient clinic for routine follow-up. On examination, she was short of breath and reported increasing orthopnea. She was admitted to the ICU for right heart catheterization. Initial readings show a cardiac index of 1.8 L/min/m2 , elevated pulmonary capillary wedge pressure (25 mmHg), and high pulmonary pressures (72/45 mmHg). Her initial BP was 105/55 mmHg, and she had a heart rate of 105 beats per minute (bpm). Home medications include captopril, spironolactone, metoprolol XL, and furosemide. Which of the following inotropic agents would be most appropriate?

A. Dopamine
B. Dobutamine
C. Milrinone
D. Isoproterenol


Question 17# Print Question

Which of the following statements is true regarding vitamin K administration?

A. Subcutaneously administered vitamin K exhibits the same bioavailability as oral or IV vitamin K
B. IV vitamin K is superior at lowering the INR than oral vitamin K at similar doses
C. IV vitamin K works faster to lower the INR than oral vitamin K at similar doses
D. Rates of anaphylaxis are similar between oral and IV administration of vitamin K


Question 18# Print Question

Which of the following agents bind only to factor Xa?

A. Enoxaparin
B. Fondaparinux
C. Bivalirudin
D. Unfractionated heparin (UFH)


Question 19# Print Question

A. F. is a 52-year-old man with a history of AFib, transient ischemic attacks, hypertension (HTN), and rheumatic heart disease. The recommendations from the Sixth American College of Chest Physicians (ACCP) Consensus Conference on Antithrombotic Therapy suggest that this patient be initiated on _____ for antithrombotic therapy because of AFib.

A. aspirin, 81 mg daily
B. aspirin, 325 mg daily
C. warfarin, with a target goal international normalized ratio (INR) of 2.5
D. warfarin, with a target goal INR of 3.5


Question 20# Print Question

The patient above is going to be electively cardioverted. What is the timing of PO anticoagulant therapy?

A. Warfarin with a target INR of 3.5 for 4 weeks before cardioversion and continued for 6 weeks after cardioversion
B. Warfarin with a target INR of 3.5 for 3 weeks before cardioversion and continued for 6 weeks after cardioversion
C. Warfarin with a target INR of 2.5 for 3 weeks before cardioversion and continued for 4 weeks after cardioversion
D. Warfarin with a target INR of 2.5 for 6 weeks before cardioversion and continued for 6 weeks after cardioversion




Category: Cardiology--->Pharmacology
Page: 4 of 11